Difference between revisions of "2017 AMC 12B Problems/Problem 20"
(Created page with "==Problem 20== Real numbers <math>x</math> and <math>y</math> are chosen independently and uniformly at random from the interval <math>(0,1)</math>. What is the probability th...") |
|||
Line 5: | Line 5: | ||
==Solution== | ==Solution== | ||
+ | <math>\lfloor\log_2x\rfloor=-1</math> when <math>1/2<x<1</math>. The chance that <math>\lfloor\log_2x\rfloor=\lfloor\log_2y\rfloor=-1</math> is <math>1/2*1/2=1/4</math>. | ||
+ | <math>\lfloor\log_2x\rfloor=-2</math> when <math>1/4<x<1/2</math>. The chance that <math>\lfloor\log_2x\rfloor=\lfloor\log_2y\rfloor=-2</math> is <math>1/4*1/4=1/16</math>. | ||
+ | This creates an infinite series, where the common ratio is <math>1/4</math>. The solution is thus <math>1/4 * 1/(1-1/4)=D: 1/3</math> |
Revision as of 20:49, 16 February 2017
Problem 20
Real numbers and are chosen independently and uniformly at random from the interval . What is the probability that , where denotes the greatest integer less than or equal to the real number ?
Solution
when . The chance that is . when . The chance that is . This creates an infinite series, where the common ratio is . The solution is thus